2014 dxdy logo

Научный форум dxdy

Математика, Физика, Computer Science, Machine Learning, LaTeX, Механика и Техника, Химия,
Биология и Медицина, Экономика и Финансовая Математика, Гуманитарные науки


Правила форума


Посмотреть правила форума



Начать новую тему Ответить на тему
 
 Ещё комплан
Сообщение18.03.2018, 17:13 


08/12/17
255
Пусть $f(z)$ голоморфна в круге $D$ с центром в начале координат и радиусом $R$, непрерывна в замыкании этого круга. $M=\max\limits_{\left\lvert z\right\rvert=R}\left\lvert f(z)\right\rvert$.
Необходимо доказать, что при $n\in \mathbb{N}\cup 0$ и $\left\lvert z\right\rvert<R$ справедливы:
1) $\left\lvert \frac{f^{(n)}(z)}{n!}\right\rvert\leqslant \frac{MR}{(R-\left\lvert z\right\rvert)^{n+1}}$
2) $\left\lvert f'(z)\right\rvert\leqslant \frac{MR}{R^2-\left\lvert z\right\rvert^2}$

Из своих мыслей лишь оценка $\left\lvert \int\limits_{\gamma}^{} f(z)d\gamma\right\rvert\leqslant M\left\lvert\gamma\right\rvert$, где $\gamma = R e^{i\varphi}$

 Профиль  
                  
 
 Re: Ещё комплан
Сообщение18.03.2018, 17:17 
Заслуженный участник
Аватара пользователя


27/12/17
1411
Антарктика
Тут надо представить $f(z)$ в виде интеграла типа Коши и вспомнить формулу его $n$-й производной

 Профиль  
                  
 
 Re: Ещё комплан
Сообщение18.03.2018, 22:32 


08/12/17
255
Обозначим $\gamma_1=R e^{it}$ окружность с центром в нуле и радиусом R,
$\gamma_2=z e^{it}$ окружность с центром в нуле и радиусом z.
$G$ - область, ограниченная этими окружностями.
Тогда $\left\lvert \frac{f^{n}(z)}{n!}\right\rvert=\left\lvert \frac{1}{2\pi i}\int\limits_{\partial G}^{}\frac{f(\zeta)d\zeta}{(\zeta-z)^{n+1}}\right\rvert=\frac{1}{2\pi}\left\lvert \int\limits_{\gamma_1}^{}\frac{f(\zeta)d\zeta}{(\zeta-z)^{n+1}}-\int\limits_{\gamma_2}^{}\frac{f(\zeta)d\zeta}{(\zeta-z)^{n+1}}\right\rvert\leqslant\frac{1}{2\pi}\left\lvert \int\limits_{\gamma_1}^{}\frac{f(\zeta)d\zeta}{(\zeta-z)^{n+1}}\right\rvert\leqslant \frac{1}{2\pi}\int\limits_{\gamma_1}^{}\left\lvert \frac{f(\zeta)}{(\zeta-z)^{n+1}}\right\rvert \left\lvert d\gamma_1\right\rvert\leqslant \frac{1}{2\pi}\frac{M}{(R-\left\lvert z\right\rvert)^{n+1}}\int\limits_{0}^{2\pi}Rdt=\frac{1}{2\pi}\frac{M}{(R-\left\lvert z\right\rvert)^{n+1}}2\pi R=\frac{MR}{(R-\left\lvert z\right\rvert)^{n+1}}$
Истина? Или где-то рядом? Или где-то там?

 Профиль  
                  
 
 Re: Ещё комплан
Сообщение19.03.2018, 03:19 
Заслуженный участник
Аватара пользователя


27/12/17
1411
Антарктика
Второй интеграл вообще ни к чему, можно сразу брать по большой окружности, т.к. интеграл типа Коши определен на любой кривой, лишь бы через точку $z$ не проходила

-- 19.03.2018, 05:22 --

А вот во втором примере, похоже, так просто не выйдет. Придется более аккуратно оценивать, т.е. сначала параметризовать интеграл, преобразовывать, а модули навешивать в последнюю очередь. По-крайней мере мне так кажется на первый взгляд.

 Профиль  
                  
 
 Re: Ещё комплан
Сообщение19.03.2018, 15:38 
Заслуженный участник
Аватара пользователя


27/12/17
1411
Антарктика
Подсказка по второй задаче (т.к. она далеко не такая простая, как первая, и я сам не сразу допёр): $\left\lvert\xi-z\right\rvert^2=(\xi-z)(\bar{\xi}-\bar{z})$, затем замена $\xi=Re^{i\varphi}$.
В итоге получается интеграл $\int\limits_{0}^{2\pi}\frac{1}{R^2+\left\lvert{z}\right\rvert^2-2R\left\lvert{z}\right\rvert\cos{\psi}}d\psi$ (буква $\psi$ написана не случайно), ну и затем этот интеграл надо посчитать. Вот его значение и даст нужный ответ. Просто оценками тут не отделаться.

 Профиль  
                  
 
 Re: Ещё комплан
Сообщение19.03.2018, 19:53 


08/12/17
255
$\psi=\varphi+\alpha$, где $\alpha=\arg \frac{\overline{z}}{\left\lvert z\right\rvert}$. Верно расшифровал?

 Профиль  
                  
 
 Re: Ещё комплан
Сообщение20.03.2018, 03:19 
Заслуженный участник
Аватара пользователя


27/12/17
1411
Антарктика
Ага (правда, там косинус разности). Еще, лучше интеграл считать в пределах $[-\pi,\pi]$

 Профиль  
                  
 
 Re: Ещё комплан
Сообщение20.03.2018, 09:39 


08/12/17
255
thething в сообщении #1298457 писал(а):
пределах $[-\pi,\pi]$

У меня в итоге получается так
$\frac{4}{R^2-\left\lvert z\right\rvert^2}\arctg \frac{(R+\left\lvert z\right\rvert)\tg\frac{\psi}{2}}{R-\left\lvert z\right\rvert}$ от 0 до $\pi$, что равно $\frac{4}{R^2-\left\lvert z\right\rvert^2}(\arctg(\infty)-\arctg 0)=\frac{2\pi}{R^2-\left\lvert z\right\rvert^2}$. Насколько верно? Или как-то лучше и проще можно?

 Профиль  
                  
 
 Re: Ещё комплан
Сообщение20.03.2018, 11:04 
Заслуженный участник
Аватара пользователя


27/12/17
1411
Антарктика
Ответ-то в-общем правильный, но почему такой ответ только на половине отрезка, когда должен получиться на всем? Или Вы четность учли? Если да, то все верно. Если нет, то в каком-то месте в два раза ошиблись.

Касаемо проще -- никак особо проще не будет. Универсальная тригонометрическая подстановка, которую Вы использовали, самый простой путь.

 Профиль  
                  
 
 Re: Ещё комплан
Сообщение20.03.2018, 15:28 


08/12/17
255
thething в сообщении #1298477 писал(а):
Или Вы четность учли?

Да, учёл чётность. Поэтому удвоил интеграл. $2\pi$ - итоговый ответ.

 Профиль  
                  
 
 Re: Ещё комплан
Сообщение20.03.2018, 15:33 
Заслуженный участник
Аватара пользователя


27/12/17
1411
Антарктика
Тогда все правильно

 Профиль  
                  
Показать сообщения за:  Поле сортировки  
Начать новую тему Ответить на тему  [ Сообщений: 11 ] 

Модераторы: Модераторы Математики, Супермодераторы



Кто сейчас на конференции

Сейчас этот форум просматривают: нет зарегистрированных пользователей


Вы не можете начинать темы
Вы не можете отвечать на сообщения
Вы не можете редактировать свои сообщения
Вы не можете удалять свои сообщения
Вы не можете добавлять вложения

Найти:
Powered by phpBB © 2000, 2002, 2005, 2007 phpBB Group